m
Line 21: Line 21:
 
but haven't had any luck; I always end up with an indeterminate value relating one and infinity that I can't break down into something manageable with L'Hopital's.--[[User:Jmason|Jmason]] 15:53, 2 October 2008 (UTC)
 
but haven't had any luck; I always end up with an indeterminate value relating one and infinity that I can't break down into something manageable with L'Hopital's.--[[User:Jmason|Jmason]] 15:53, 2 October 2008 (UTC)
  
*This is actually in our book, but I'll redo the work here to practice with Latex and so you can see it here instead of going to the book.
+
*To Gary: Yeah, I have to agree with John, I'm not following your math here.  Could you tell us how you got to that equation?
 +
 
 +
To John: The derivation is actually in our book, but I'll redo the work here to practice with Latex and so you can see it here instead of going to the book.
  
 
<math>A=P\bigg(1+\frac{r}{n}\bigg)^{nt}</math>
 
<math>A=P\bigg(1+\frac{r}{n}\bigg)^{nt}</math>

Revision as of 15:38, 2 October 2008

I'm not sure that your original function makes that much sense; I can't say that I can tell how you got to that point.

Just checking out how normal compounded interest works, I checked Wikipedia and rediscovered the formula:

$ A=P\bigg(1+\frac{r}{n}\bigg)^{nt} $

P = principal amount (initial investment)

r = annual nominal interest rate (as a decimal)

n = number of times the interest is compounded per year

t = number of years

A = amount after time t

I've tried to break that down into good 'ole

$ A=Pe^{rt} $

but haven't had any luck; I always end up with an indeterminate value relating one and infinity that I can't break down into something manageable with L'Hopital's.--Jmason 15:53, 2 October 2008 (UTC)

  • To Gary: Yeah, I have to agree with John, I'm not following your math here. Could you tell us how you got to that equation?

To John: The derivation is actually in our book, but I'll redo the work here to practice with Latex and so you can see it here instead of going to the book.

$ A=P\bigg(1+\frac{r}{n}\bigg)^{nt} $

Now we we want to find the limit as n goes to infinity, since we're trying to compound the interest continuously, and therefore have an infinite number of times we compound.

$ P\lim_{n\to\infty}\bigg(1+\frac{r}{n}\bigg)^{nt} $

We now have a limit in the indeterminate form $ 1^{\infty} $

Now I'm going to stray from what the book showed us and use L'H rule to show that the limit approaches $ Pe^{rt} $

First, drop the P, we can multiply it back in later. Also, since it's in form $ 1^{\infty} $ we can try to find its limit by taking the natural log of it and its limit (Which means when we find the new limit, we have to raise e to that power to get the right limit, as you already know.)

$ \lim_{n\to\infty}\ln{\bigg(1+\frac{r}{n}\bigg)^{nt}} $

Move the nt to the front

$ \lim_{n\to\infty}nt\ln{\bigg(1+\frac{r}{n}\bigg)} $

And move the nt to the bottom by inverting it

$ \lim_{n\to\infty}\frac{\ln{\bigg(1+\frac{r}{n}\bigg)}}{\frac{1}{nt}} $

Which is now in the indeterminate form $ \frac{0}{0} $ So apply L'H rule and find derivatives of top and bottom functions:

$ \lim_{n\to\infty}\frac{\frac{-r}{(1+\frac{r}{n})n^2}}{\frac{-1}{n^2t}} $

Now the $ -n^2 $ cancel and we can take the limit as n approaches infinity.


$ \lim_{n\to\infty}\frac{rt}{1+\frac{r}{n}}=\frac{rt}{1}=rt $

Now take e to this power to get the actual limit.

$ \lim_{n\to\infty}\bigg(1+\frac{r}{n}\bigg)^{nt}=e^{rt} $

And now we simply add back in our constant to solve completely.

$ A=P\lim_{n\to\infty}\bigg(1+\frac{r}{n}\bigg)^{nt}=Pe^{rt} $ Jhunsber

Alumni Liaison

Meet a recent graduate heading to Sweden for a Postdoctorate.

Christine Berkesch